LSAT and Law School Admissions Forum

Get expert LSAT preparation and law school admissions advice from PowerScore Test Preparation.

User avatar
 Dave Killoran
PowerScore Staff
  • PowerScore Staff
  • Posts: 5852
  • Joined: Mar 25, 2011
|
#55801
Complete Question Explanation
(The complete setup for this game can be found here: lsat/viewtopic.php?t=23743)

The correct answer choice is (D)

The condition in the question stem creates the following block and sequence:

pt45_d04_g1_q4.png


From this sequence, it is obvious that Patterson cannot meet with Y any later than 4:00 because Patterson must meet with W and T later than with Y. Consequently, answer choice (D) is correct.

Answer choice (A): This answer choice is incorrect. Patterson can meet with R at 2:00, as shown by the following hypothetical: U-R-S-Y-W-T.

Answer choice (B): This answer choice is incorrect. Patterson can meet with Y at 3:00, as shown by the following hypothetical: U-S-Y-W-T-R.

Answer choice (C): This answer choice is incorrect. Patterson can meet with T at 4:00, as shown by the following hypothetical: S-Y-W-T-U-R.

Answer choice (D): This is the correct answer choice.

Answer choice (E): This answer choice is incorrect. Patterson can meet with T at 6:00, as shown by the following hypothetical: U-R-S-Y-W-T.
You do not have the required permissions to view the files attached to this post.

Get the most out of your LSAT Prep Plus subscription.

Analyze and track your performance with our Testing and Analytics Package.